Difference between revisions of "1981 AHSME Problems/Problem 22"
m |
J314andrews (talk | contribs) m (→Solution 1) |
||
(6 intermediate revisions by 3 users not shown) | |||
Line 1: | Line 1: | ||
− | ==Problem== | + | ==Problem 22== |
How many lines in a three dimensional rectangular coordinate system pass through four distinct points of the form <math>(i, j, k)</math>, where <math>i</math>, <math>j</math>, and <math>k</math> are positive integers not exceeding four? | How many lines in a three dimensional rectangular coordinate system pass through four distinct points of the form <math>(i, j, k)</math>, where <math>i</math>, <math>j</math>, and <math>k</math> are positive integers not exceeding four? | ||
<math>\textbf{(A)}\ 60\qquad\textbf{(B)}\ 64\qquad\textbf{(C)}\ 72\qquad\textbf{(D)}\ 76\qquad\textbf{(E)}\ 100</math> | <math>\textbf{(A)}\ 60\qquad\textbf{(B)}\ 64\qquad\textbf{(C)}\ 72\qquad\textbf{(D)}\ 76\qquad\textbf{(E)}\ 100</math> | ||
− | ==Solution== | + | ==Solution 1== |
− | + | ||
+ | Let <math>(i, j, k)</math> be the first point whose coordinates are positive integers at most <math>4</math> that a line passes through when being traced in a certain direction. Then the next three lattice points the line passes through must be in the form <math>(i + a, j + b, k + c)</math>, <math>(i + 2a, j + 2b, k + 2c)</math>, and <math>(i + 3a, j + 3b, k + 3c)</math>, where <math>a, b, c</math> are integers. | ||
+ | |||
+ | Note that if <math>a \geq 2</math>, <math>i + 3a \geq 1 + 3 \cdot 2 = 7</math>, which is too large. Therefore <math>a \leq 1</math>, and by similar logic <math>b \leq 1</math> and <math>c \leq 1</math>. Also, if <math>a \leq -2</math>, <math>i + 3a \leq 4 + 3(-2) = -2</math>, which is too small. Therefore, <math>a \geq -1</math>, and by similar logic <math>b \geq -1</math> and <math>c \geq -1</math>. So <math>a, b, c \in \{-1, 0, 1\}</math>. | ||
+ | |||
+ | If <math>a = 1</math>, then <math>1 \leq i, i+1, i+2, i+3 \leq 4</math>. In this case, only <math>i = 1</math> is valid. | ||
+ | |||
+ | If <math>a = 0</math>, then <math>1 \leq i \leq 4</math>. In this case, <math>i = 1, 2, 3, 4</math> are all valid. | ||
+ | |||
+ | If <math>a = -1</math>, then <math>1 \leq i, i-1, i-2, i-3 \leq 4</math>. In this case, only <math>i = 4</math> is valid. | ||
+ | |||
+ | Therefore, <math>(a, i) \in \{(1, 1), (0, 1), (0, 2), (0, 3), (0, 4), (-1, 4)\}</math>. By similar logic, <math>(b, j)</math> and <math>(c, k)</math> must also be in this set. | ||
+ | |||
+ | If <math>a = b = c = 0</math>, then all four points are <math>(i, j, k)</math>, so at least one of <math>a, b, c</math> must be nonzero. Therefore, there are <math>6^3 - 4^3 = 216 - 64 = 152</math> choices for <math>(i, j, k, a, b, c)</math>. However, each line can be determined by two different values of <math>(i, j, k, a, b, c)</math>, as the line can be traced in two different directions. For instance, <math>(i, j, k, a, b, c) = (4, 1, 3, -1, 1, 0)</math> and <math>(i, j, k, a, b, c) = (1, 4, 3, 1, -1, 0)</math> determine the line containing <math>(4, 1, 3), (3, 2, 3), (2, 3, 3), (1, 4, 3)</math>. Therefore, there are <math>\frac{152}{2} = \boxed{(\textbf{D})\ 76}</math> such lines. | ||
+ | |||
+ | ==See also== | ||
+ | |||
+ | {{AHSME box|year=1981|num-b=21|num-a=23}} | ||
+ | {{MAA Notice}} |
Latest revision as of 13:32, 20 August 2025
Problem 22
How many lines in a three dimensional rectangular coordinate system pass through four distinct points of the form , where
,
, and
are positive integers not exceeding four?
Solution 1
Let be the first point whose coordinates are positive integers at most
that a line passes through when being traced in a certain direction. Then the next three lattice points the line passes through must be in the form
,
, and
, where
are integers.
Note that if ,
, which is too large. Therefore
, and by similar logic
and
. Also, if
,
, which is too small. Therefore,
, and by similar logic
and
. So
.
If , then
. In this case, only
is valid.
If , then
. In this case,
are all valid.
If , then
. In this case, only
is valid.
Therefore, . By similar logic,
and
must also be in this set.
If , then all four points are
, so at least one of
must be nonzero. Therefore, there are
choices for
. However, each line can be determined by two different values of
, as the line can be traced in two different directions. For instance,
and
determine the line containing
. Therefore, there are
such lines.
See also
1981 AHSME (Problems • Answer Key • Resources) | ||
Preceded by Problem 21 |
Followed by Problem 23 | |
1 • 2 • 3 • 4 • 5 • 6 • 7 • 8 • 9 • 10 • 11 • 12 • 13 • 14 • 15 • 16 • 17 • 18 • 19 • 20 • 21 • 22 • 23 • 24 • 25 • 26 • 27 • 28 • 29 • 30 | ||
All AHSME Problems and Solutions |
These problems are copyrighted © by the Mathematical Association of America, as part of the American Mathematics Competitions.